Side 1 av 1

Vis at f(a) = a^3 er strengt voksende

Lagt inn: 26/10-2019 20:42
av Oskaroskar
Jeg aner ikke hvordan jeg skal gå frem her så håper jeg kan få litt hjelp.
Fasiten sier at man skal ta
(a1)^3 - (a2)^3 (1)
= (a1 - a2)( (a1)^2 + a1a2 + (a2)^2) (2)
= (a1 - a2) [((a1) + 1/2(a2))^2 + 3/4(a2)^2] (3)

Der a1 og a2 er nedfelte; a(nedfelt 1 el. 2) ikke multipl!

Jeg trenger en forklaring på hvordan man konkluderer med å ta (a1)^3 - (a2)^3 i utgansgpunktet ,hvordan man kommer fra 1 til 2 og hvordan man kommer fra 2 til 3. Her skjønner jeg lite, og spesielt hvordam man kommer fram til (3/4(a2))^2

I tillegg konkluderes det med at f er strengt voksende fordi hakeparantesen er positiv. Dette trenger jeg også forklart da uttrykket før a1 -a2 er negativt

Håper noen kan hjelpe. Takk :)

Re: Vis at f(a) = a^3 er strengt voksende

Lagt inn: 26/10-2019 21:02
av Aleks855
Det er litt uklart hva som menes med $a_1$ og $a_2$ her. Er det noe som refereres til i oppgaveteksten?

Oppgaven kan forøvrig løses ved å betrakte at $f'(a) = 3a^2 \geq 0$ for alle $a\in\mathbb R$ som viser at funksjonen er voksende overalt, bortsett fra det ene flate punktet $a=0$.

Re: Vis at f(a) = a^3 er strengt voksende

Lagt inn: 26/10-2019 23:00
av Oskaroskar
Konkrete oppgaven er: vis at x^3 er strengt voksende.
Hint; (x1)^3 - (x2)^3 = ((x1) - (x2))((x1)^2 + (x1)(x2) + (x2)^2)
= ((x1) - (x2)) [((x1) + 1/2(x2))^2 + 3/4(x2)^2]

Vet ikke om det ble forvirrende at jeg bruke a isteden for x, ville ikke tså det skulle misforstås med multipl.

Jeg ville også vist det med x^3 = 3x^2 som ikke kan være neg. og evt tatt f(x+h) - f(a) for å komme frem til det slik som man lærer i delkap. Men boka vil noe mer

Re: Vis at f(a) = a^3 er strengt voksende

Lagt inn: 28/10-2019 12:09
av DennisChristensen
Anta at $b>a$. Vi ønsker å vise at $f(b) > f(a)$. Resultatet er trivielt hvis $a=0$ eller $b=0$, så anta at $a\neq 0\neq b$. Da har vi at
$$f(b) - f(a) = b^3 - a^3 = (b - a)(a^2 + ab + b^2) > (b - a)(a^2 - 2|ab| + b^2) = (b-a)(|a| - |b|)^2 \geq 0,$$
så $f$ er strengt voksende.

Re: Vis at f(a) = a^3 er strengt voksende

Lagt inn: 28/10-2019 15:59
av Nebuchadnezzar
Dette ble vel ikke helt riktig Dennis. Du ønsker å vise at $b>a$ medfører $f(b)>f(a)$, men det du viser er jo at $f(b) \geq f(a)$. Altså er ikke $x^3$ en strengt voksende funksjon (problemet er jo som Aleks sier i null).

Så her må du nok begrense definisjonsmengden til funksjonen til noe mindre enn $\mathbb{R}$ for at beviset ditt skal stemme =)

Re: Vis at f(a) = a^3 er strengt voksende

Lagt inn: 28/10-2019 23:00
av DennisChristensen
Nebuchadnezzar skrev:Dette ble vel ikke helt riktig Dennis. Du ønsker å vise at $b>a$ medfører $f(b)>f(a)$, men det du viser er jo at $f(b) \geq f(a)$.
Feil. Beviset mitt viser streng ulikhet. Nøkkelen ligger i steget $(b-a)(a^2 +ab+b^2) > (b-a)(a^2 - 2|ab|+b^2)$ som gjelder for alle $a\neq 0\neq b$ hvor $b>a$ (og tilfellet hvor $a=0$ eller $b=0$ er allerede betraktet).
Nebuchadnezzar skrev: Altså er ikke $x^3$ en strengt voksende funksjon (problemet er jo som Aleks sier i null).
Også feil. Det er fullt mulig for en strengt voksende funksjon å ha $0$ som derivert i et punkt, som jo beviset illustrerer.

Re: Vis at f(a) = a^3 er strengt voksende

Lagt inn: 29/10-2019 13:50
av Nebuchadnezzar
Ja, her legger jeg meg flat. Litt lenge siden jeg hadde om dette i matte 1.